Ashley plays in the band. She brought eighty-four snacks to school for her band friends and she has twenty-one friends. How many snacks does each person receive?

Ashley Plays In The Band. She Brought Eighty-four Snacks To School For Her Band Friends And She Has Twenty-one

Answers

Answer 1

Answer:

B. 4 steps

Step-by-step explanation:

[tex]84 \div 21 = 4[/tex]

just divide total number of steps by number of friends to derive the answer.
Answer 2
given:

84 snacks

21 friends

to find:

how many snacks each of her friend gets.

solution:

[tex] \frac{84}{21} [/tex]

[tex] = 4[/tex]

hence, each of her friend will get 4 snacks.

answer= option B


Related Questions

Read the scenario. 1. The temperature at 6 A.M. was -5°F. By noon the temperature had risen by 5°F. What was the temperature at noon? Represent these values with integers and model them on a number line. + + + 0 1 2 3 4 5 6 7 8 9 10 -10 -9 -8 -7 -6 -5 -4 -3 -2 -1 Read the scenario. 2. A diver was 8 feet below sea level. The diver swam back up 8 feet. What is the location of the diver? Represent these values with integers and model them using integer tiles.​

Answers

Answer:

mm

Step-by-step explanation:

n is an integer greater than 1
Prove algebraically that 10+ n^2 - (n - 2)^2 is always an even number.
Your final line must have, 'always even' as part of the line.

Answers

Answer:

Proof below.

Step-by-step explanation:

Let f(n)=10+ n^2 - (n - 2)^2.

We want to prove f is even for integer n > or = 1.

Let's show f is even for the base case, n=1.

f(1)=10+1^2-(1-2)^2

f(1)=10+1-(-1)^2

f(1)=11-(1)

f(1)=10

f(1) is even.

Let's assume the for some integer k > or = 1 , we have f(k) is even. This means there is some integer m so that f(k)=2m.

Note: f(k)=10+ k^2 - (k - 2)^2.

We want to show f(k+1) is even.

f(k+1)=10+ (k+1)^2 - ([k+1] - 2)^2

f(k+1)=10+ (k+1)^2 - (k-1)^2

f(k+1)=10+ ((k+1)^2 - (k-1)^2)

That difference is a difference of squares. Let's try factoring it.

f(k+1)=10+ (k+1-[k-1])(k+1+[k-1])

f(k+1)=10+ (2)(2k)

Multiply:

f(k+1)=10+4k

Turns out we didn't even need to use our induction hypothesis since 10+4k is 2(5+2k) which makes f(k+1) even given the factor of 2 present.

This proves for all integers n>=1 that f(n) is even.

We could have this without induction then. Let's do that here below this line.

-------------------

10+ n^2 - (n - 2)^2

10+[n^2 - (n - 2)^2]

Factor difference of squares:

10+(n-(n-2))(n+(n-2))

Simplify:

10+(2)(2n-2)

Factor:

2[5+(2n-2)]

Because of the factor of 2, the given expression is always even for integer n.

Find the circumference of the
circle. Use 3.14 for it. Do not
round your answer.
C = [ ? ] feet
18 feet
Enter the number that
belongs in the green box.
Hint: C = 2tr

Answers

The answer is abc deft hi

Simplify x+2x –8+3x+1 –2x.

Answers

Answer:

4x - 7

Step-by-step explanation:

x + 2x + 3x - 2x = 4x

-8 + 1 = -7

The simplified expression is 4x -7.

What is Expression?

A fine operation similar as deduction, addition, addition, or division is used to combine terms into an expression. In a fine expression, the following terms are used.

An absolute numerical number is appertained to as a constant.Variable- A symbol without a set value is appertained to as a variable. Term: A term can be made up of a single constant, a single variable, or a blend of variables and constants multiplied or divided.Coefficient In an expression, a measure is a number that's multiplied by a variable.

Given:

Expression:  x+2x –8+3x+1 –2x

Now, simplifying the expression by taking like terms along.

x+2x –8+3x+1 –2x

= x+ 2x - 2x + 3x - 8 + 1

= x(1 + 2 - 2 + 3) -7

= x( 4) -7

= 4x -7

Learn more about expression here:

https://brainly.com/question/14083225

#SPJ2

someone please help me

Answers

Answer:

  (a)  A is steeper; 4/5 > 2/5

Step-by-step explanation:

The slope of A is the coefficient of x, so is 4/5. This eliminates choices B and D.

The line in the graph has a rise of 2 units for each run of 5 units, so has a slope of ...

  slope = rise/run = 2/5

This eliminates choice C.

The correct description of the relationship is the first one:

  Equation A has a steeper slope, because 4/5 is greater than 2/5.

What are the sums for both of those?

Answers

I don’t think I can help you with this but good luck! And I hope you have an amazing day! :)

Someone plz help me :(

Answers

Step-by-step explanation:

option A is correct answer

Answer:

Should be C if not then A

Step-by-step explanation:

Please help!! Thank you!! :)

Answers

Answer:

the answer for this equation is -32

3-7(-5)
38
Plug the numbers into the equation and ta da :)

the graph shown below expresses a radical function that can be written in the form f(x)=a(x+k)^1/n+c. what does the graph tell you about the value of c in this function?

Answers

Answer:

c = 2.

Step-by-step explanation:

When the first part of the function has the value 0,  f(x) will also be 0 if the c wasn't there.

So the value of c is 2.

From the given graph n is a positive odd number.

What is the exponential function?

Exponential function, as its name suggests, involves exponents. But note that, an exponential function has a constant as its base and a variable as its exponent but not the other way round (if a function has a variable as the base and a constant as the exponent then it is a power function but not an exponential function).

From the graph,

All the values of the graph are positive values (even for the negative values of x), but in the left side we can see that the function decreases and is different than the right side.

So this is not an even function, then n must be an odd number (n odd allows us to have negative values for y = f(x) that happen when x + k is negative).

Also, we can see that the function increases, if n was a negative number, like: n = -N

We would have: [tex]f(x)=\frac{a}{(x+k)^{\frac{1}{N} }} +c[/tex]

So in this case x is in the denominator, so as x increases, we would see that the value of y decreases, but that does not happen, so we can conclude that the value of n must be positive.

Then n is a positive odd number.

Therefore, n is a positive odd number.

Learn more about the exponential function here:

brainly.com/question/11487261.

#SPJ7

Find the perimeter. Simplify your answer.

Answers

7y+10+7y+10+y-4

14y+20+y-4

15y+20-4

15y+16

answer= 15y+16

Answer:

7y+10+7y+10+y-4=

(7y+7y+y)+(10+10-4)=

15y+16=

If a company had a gain of 800$ in March , but in April and May , it had a loss of $400 and 900$ respectively , how mich profit or loss did the company have in three months .
Please answer quickly ,.I will mark you as brainlist ... In Hurry Plz

Answers

500 is the answer
500

Answer:

-500

Step-by-step explanation:

i knew the answer

have a great day ;)

solve the inequality 4x-3=2x+7

Answers

Answer: x=-2

Step-by-step explanation:

Add three to both sides of the equation

4x-3=2x+7

4−3+3=2−7+3

simplify

4=2−4

4−2=2x-4−2

simplify

2=−4

Divide both sides of the equation by the same term

2=−4

2/2=−4/2

=−2

Answer: x = 5

Step-by-step explanation:

Isolate the variable by dividing each side by factors that don't contain the variable.x=5

What is the measurement of this angle?

Answers

The measurement of all angles is 180, so just subtract the other two angles from 180

180 - 59 - 67 = 54 degrees

Answer:

54°

Step-by-step explanation:

All the angle measures of a triangle must add up to equal 180°. So,

59°+67°+x°=180°

x=54°

6.
3. Use the clues to determine
the number
I have 36 hundreds, 34 tens, and
7 ones. What number am I?

Answers

The number is 3947

It should be noted that 36 hundreds will be equal to:

= 36 × 100

= 3600

It should be noted that 34 tens will be:

= 34 × 10

= 340

7 ones will be:

= 7 × 1

= 7.

Therefore, the sum will be:

= 3600 + 340 + 7 = 3947

Answer:

3947

Step-by-step explanation:

help me brain pt Part 5

Answers

Answer is A. Adjacent

help.. What is the domain of the function

Answers

Answer:

i think d

Step-by-step explanation:

The force, f acting on a chraged object varies inversley to the square of its distance from another charged object. when the two objects are 0.64 meters apart, the force acting on them is 8.2 netwons

Answers

Btw also found a spelling mistake in the question:instead of newtons you put netwons-      lol-

btw  

Rounded to the nearest thousandth would be 4.444 Newtons (same units you solved for k with)

But anyway here you go:

~Jamie

Jason was a recent graduate from a university program with a Bachelor of Science in Medical Technology. He will begin a position at the local hospital working 40 hours a week with an hourly rate of $25.50. He will receive a paycheck every two weeks. In a document, calculate the following. Be sure to show your mathematical calculations.
Gross Pay for 1 paycheck
Deductions for federal tax, state tax, and city tax
Deduction for F.I.C.A. or social security tax
Deduction of $117 for medical and dental insurance.
What would Jason’s net monthly income and net annual salary be after taxes and insurance?

Answers

Answer:

Gross Pay for 1 paycheck

Deductions for federal tax, state tax, and city tax hope it helps :)

Step-by-step explanation:

The net monthly income of Jason is $194.68 and the net annual salary is 23324.16.

What is gross pay?

Gross pay of an employee is basically sum of his basic pay, dearness allowance, any other allowance only. Public providend fund, income tax are not deducted from it.

How to find net pay?

1) Gross paycheck=2*40*25.5=$2040.

2) Federal tax=2040*21%=$428.4

3) State tax=25=$25

4) City tax=2040*12%=$244.8.

5) Deductions=2040*12.45=$252.96.

6) Net monthly income=2*(Gross pay-federal tax-state tax-city tax-deductions-deductions for F.IC.A. or social security tax)

=2*[2040-428.4-25-244.8-252.96-117]

=1943.68

7) Net annual salary=12* net monthly income.

=1943.68*12  ( 1year= 12 months)

=$23324.16

Hence the net monthly income of Jason is $194.68 and the net annual salary is 23324.16.

Learn more about gross pay at https://brainly.com/question/11274742

#SPJ2

Use the long division method to find the result when 6x^3+ 23x^2 + 15x + 28 is
divided by 2x + 7

Answers

The answer is:

3x^3+x+4.

Hope this helped!! :)

help on #8 please??
question is in picture

Answers

Answer:

x = 6

Step-by-step explanation:

To get rid of the fraction at the left side of the equation, multiply both sides by 4

20x + 60 = 180 now subtract 60 from both sides

20x = 120 divide both sides by 20

x = 6

8b–9b–5b+7b
Simplify the expression:

Answers

Answer:

b, or 1b

Step-by-step explanation:

First of all, you have to combine like terms. In this situation though, they are all b. So start at the beginning. 8-9 is -1. -1 - 5 is -6. -6+7 is 1. So you say, b, because it is the same thing as 1 b.

PARALLEL PERPENDICULAR OR NEITHER OR SAME LINE

Answers

Answer:

parallel, perpendicular are opposite

Answer:

PARALLEL PERPENDICULAR

Step-by-step explanation:

Find the measure of each marked angle.

Answers

Answer:

73 and 73

Step-by-step explanation:

2x+45=5x+3

3x=42

x=14

2(14)+45 =73

the other should also be 73

When ordering pizzas, 5 pizzas cost $60. Does this mean 9 pizzas will cost $100? Write a proportion and use cross products.

Answers

Answer:

108$.

Step-by-step explanation:

A man Buy television set at a price exclusive of sales tax for $2124 if sales tax of 12% is changed how much money does the man pay

Answers

Answer:

2378.88

Step-by-step explanation:

12/100 x 2124

=254.88

2124+254.88=answer

6457/25 using partial products

Set up using the same way please.

Answers

Answer: 6457/25=258.28

Step-by-step explanation:

Answer:

258.28  Hope this helps! Please mark brainliest it means a lot to me

Step-by-step explanation:

Th=0

H=2

T=5

0=8

tenth=2

hundredth=8

You could get the answer by doing long division/standard algorithm

Any help would be appreciated ​

Answers

Answer:

10

Step-by-step explanation:

Bisect the 72° angle to get an exterior angle of 36°.

The sum of the measures of the exterior angles of a polygon, one per vertex, is 360°.

360°/36° = 10

Answer: There are 10 sides.

can someone help me on this one plss i REALLY need this for school

Answers

Answer:

0.9 + n = 5.2

Step-by-step explanation:

Think about it, 0.9 increased by a number is 5.2

In other words, increasing 0.9 by a number will equal 5.2

So, increasing = adding, right?

Adding 0.9 to a number will equal 5.2

0.9 + n (a number) = 5.2

C increased equals +

answer..............................

Answers

Answer:

yes

Step-by-step explanation:

The proof shows that opposite angles of a parallelogram are congruent. Given: ABCD is a parallelogram with diagonal AC. Prove: ∠ BAD ≅ ∠ DCB What is the missing reason In this partial proof? Question 3 options:
ASA Substution Angle Angle Postulate Alternate Interior Angles are Congruent

Answers

A parallelogram is a figure which has its opposite sides to be equal and parallel. The missing reason in the proof is:

B. Substitution Angle Angle Postulate.

A parallelogram is a type of quadrilateral that has its opposite sides to be equal and parallel. The sum of its internal angles is [tex]360^{o}[/tex].

To prove that ∠ BAD ≅ ∠ DCB, we have:

Given parallelogram ABCD;

<BAC ≅ <ACD (alternate angle theorem)

<DAC ≅ <ACB (alternate angle theorem)

<BAC + <DAC = <BAD

Also,

<BCA + <DCA = <BCD

Therefore,

<BAD ≅ <DCB (Substitution Angle Angle Postulate)

Thus, the missing reason in the partial proof is:

option B. Substitution Angle Angle Postulate

A sketch is attached to this question for more clarifications.

Visit: https://brainly.com/question/3100335

Other Questions
Hoping to lure more shoppers downtown, a city builds a new public parking garage in the central business district. The city plans to pay for the structure through parking fees. During a two-month period (44 weekdays), daily fees collected averaged $126, with a standard deviation of $15. a) What assumptions must you make in order to use these statistics for inference 2.Mt cng ty s bn N quyn sch sau khi chi tiu x nghn $ cho vic qung co quyn schmi ny, c cho bi2 N x x x = 60 5 30 8m 5 for m = 9 answer please CAN YALL GO FOLLOW MY INS PLSS yeabdagod Ty, who thinks that he is the handsomest man in his city, feels superior to everyone around him. He is also sure that everyone knows how inferior they are compared to him. He is MOST likely experiencing: Which statement is true?A) Characters in a story can only be people.B) Animals can be characters in a story.C) Only animals can be characters in a story.D) Neither people nor animals can be characters in a story. --------is the study of how muscles work. what can happen if a person under 21 is car drinking by a law enforcement officer? Many interest groups employ __________, who are hired to represent the groups' interests to the government. A process in which cells surround and engulf substances that are too big to passthrough the membrane is known as...membrane pumpendocytosisfacilitated diffusionexocytosis When fencing with a sabre, thrusts or cuts on any part of the body earn points during right-of-attack.A. true B. false One who believes that some living things have greater moral status than others is known as a species ________. Civil law is thus distinguished from the law ofnations."Which of the following words could replace theunderlined words in the sentence?A. religious, differentB. national, combinedC. military, comparedD. governmental, separatedExte26 Nstatement best summarizes the Use the binomial formula to find the coefficient of the q^4 p^17term in the expansion of (29+p)^21 what are the influences of nature and nurture pertaining to human behavior, which one is the largest contributor of behavior, nurture or nature? Can somebody plz report and ban CARDAA13 and CARIZASHINE Given the graph of y=f(x), shown as a green curve, drag the green movable points to draw the graph of y=f(x). When the green line is moved a red dashed line will appear where the original graph appeared for reference. Notice that you can control the positioning of the reflective function with the coordinate labeled "Drag Function" and control the width of the reflection with the coordinate labeled "Control Width." what effect did Anglo explorers have on the American Indians by Plymouth area? ok so....the numbers to the code are 0 3 3 8 8the number is less than 50the value of the hundredths digit is 10 times the value of the thousandths digitthe value of the ones digit is one-tenth the value of the tens digit Using your understanding of how conflict occurs, complete the sentence.The two managers often engaged in ______________________; in todays meeting, they had a spirited debate over the projects optimal workflow, and their disagreement ultimately led to a creative solution.a. relationship conflictb. groupthinkc. task conflict